Q7

 
cbravo2963
Thanks Received: 0
Vinny Gambini
Vinny Gambini
 
Posts: 1
Joined: June 05th, 2014
 
 
 

Q7

by cbravo2963 Sun Jul 06, 2014 3:03 pm

I am stuck on this question. I was able to eliminate A,C,E but for some reason I am coming up with both B and D for answers which CANNOT be true.

Can someone explain how to get the right answer, please?
 
christine.defenbaugh
Thanks Received: 585
Atticus Finch
Atticus Finch
 
Posts: 536
Joined: May 17th, 2013
 
 
 

Re: Q7

by christine.defenbaugh Mon Jul 14, 2014 9:51 pm

Thanks for posting, cbravo2963!

So, since this is a conditional question, the very first thing we should do is input the conditional information into a new sketch of our diagram:

Image

Next, we should look at how this impacts the rules we've already sketched out, and see if we can follow the inference chain. Since L's second aisle is last, we don't need to worry about the O--L rule. And since the first K is clearly aisle 2, we can cut our relative ordering cluster down to something a little simpler:

Image

Now, we have a chain of 4 elements, MKM--O, and only 6 spaces. There's only so many places that can really go! Making it worse, MKM is a chunk. The only way to fit that whole chunk in AND leave room for O after it would be if it were 3-4-5, 4-5-6, or 5-6-7.

If MKM were 3-4-5, then O would HAVE to go in 7. Anywhere else O went would require J and J to be consecutive!

Image

If MKM were 4-5-6, similarly, O would HAVE to go in 8. Anywhere else O went would require J and J to be consecutive!

Image

Now that we come to it, if MKM were 5-6-7, that would force O to be 8, and then J and J would be consecutive! Okay, so that won't work. Looks like we only have the two options above.

From the first scenario, we can see that K could indeed stock only even numbered aisles (2 and 4)! So (D) is a could be true.

In fact, all four of the wrong answers are seen in one of these two scenarios:
    (A) J is on aisle 3 in Scenario 2!
    (C) J-O-J happens in Scenario 1!
    (D) K is on even aisles in Scenario 1!
    (E) O-L happens in Scenario 2!


Only (B) must be false. O isn't on aisle 6 in either of those scenarios. If we tried to force it, that would require MKM to be in 3-4-5. That leaves only aisles 7 and 8 for the remaining two Js, and that would mean they are consecutive!

Image

The biggest culprit of incorrect answers on conditional questions is failure to fully play out the inference chain!

Please let me know if that completely answers your question!